Difference between revisions of "1986 AJHSME Problems/Problem 4"

(Solution)
(Solution)
 
Line 6: Line 6:
  
 
==Solution==
 
==Solution==
 +
<center>
 +
First, solve the sum of \( 40.3 \) and \(.07\)
 +
 +
\( \Rightarrow \) \( 40.3 + .07 = 40.37 \)
 +
 +
Multiply it with \( 1.8 \)
 +
 +
\( \Rightarrow \) \( (1.8)(40.37) \)
 +
 +
Evalutate the result
 +
 +
\( \Rightarrow \) \( 72.666 \)
 +
 +
</center>
 +
 +
Since we found the answer to be \( 72.666 \), the closest option is <math>\boxed{\text{C) 74}}</math>.
 +
 +
Therefore, the answer is <math>\boxed{\text{C}}</math>.
 +
 +
~ Solution by abcde26
 +
==Better Solution==
  
 
<center>
 
<center>

Latest revision as of 02:53, 23 October 2025

Problem

The product $(1.8)(40.3+.07)$ is closest to

$\text{(A)}\ 7 \qquad \text{(B)}\ 42 \qquad \text{(C)}\ 74 \qquad \text{(D)}\ 84 \qquad \text{(E)}\ 737$

Solution

First, solve the sum of \( 40.3 \) and \(.07\)

\( \Rightarrow \) \( 40.3 + .07 = 40.37 \)

Multiply it with \( 1.8 \)

\( \Rightarrow \) \( (1.8)(40.37) \)

Evalutate the result

\( \Rightarrow \) \( 72.666 \)

Since we found the answer to be \( 72.666 \), the closest option is $\boxed{\text{C) 74}}$.

Therefore, the answer is $\boxed{\text{C}}$.

~ Solution by abcde26

Better Solution

$(1.8)(40.37)\approx (1.8)(40)=72.$

Approximating $40.37$ instead of $1.8$ is more effective because larger numbers are less affected by absolute changes (e.g $1001$ is much closer relatively to $1000$ than $2$ is to $1$). $74$ is the closest to $72$, so the answer is $\boxed{\text{C}}$.

See Also

1986 AJHSME (ProblemsAnswer KeyResources)
Preceded by
Problem 3
Followed by
Problem 5
1 2 3 4 5 6 7 8 9 10 11 12 13 14 15 16 17 18 19 20 21 22 23 24 25
All AJHSME/AMC 8 Problems and Solutions

These problems are copyrighted © by the Mathematical Association of America, as part of the American Mathematics Competitions. AMC Logo.png